For the periodic sequence, is there always an algebraic closed form?












2












$begingroup$


This question is a generalized form of the problem I asked before:



Algebraic Closed Form for $sum_{n=1}^{k}left( n- 3 lfloor frac{n-1}{3} rfloorright)$




Let, look at this periodic sequence:



$$a_n=left{a_1,a_2,a_3,a_4,a_5,cdots a_k ; a_1,a_2,a_3,a_4,a_5, cdots a_k; a_1,a_2,a_3,a_4,a_5,cdots a_k;cdots right}$$, where $left{a_1, a_2, a_3, cdots a_k right} in mathbb{Z^{+}}$ and $a_{k+1}=a_1, a_{k+2}=a_2, a_{k+3}=a_3, cdots a_{2k}=a_k, cdots$




For the sequence $a_k=left{ a_1,a_2,a_3,...,a_k right}$ , $k$ is a finite number. $a_1,a_2,...a_k$ are arbitary numbers.



Verbally, the series $a_n$ consists of an infinite number of periodic repetitions of the finite series $a_k.$




Finally my question is:



a) If there is an exist a algebraic closed form, for finite series $a_k$, in this case, does the $ a_n $ series always have a algebraic closed form?



b) If there is not an exist a algebraic closed form, for finite series $a_k$, in this case, does the $ a_n $ series always have a algebraic closed form?




I mean ,for example:




a)



$a_n=left{ 1,3,5,7,1,3,5,7,1,3,5,7,1,3,5,7,1,3,5,7cdotsright}$



b)



$a_n=left{1,8,2,6,5,9,1,8,2,6,5,9,1,8,2,6,5,9,1,8,2,6,5,9cdotsright}$




Thank you very much.










share|cite|improve this question











$endgroup$












  • $begingroup$
    A finite sequence always does have a closed form, and so does a periodic sequence.
    $endgroup$
    – Robert Israel
    Jan 8 at 12:39










  • $begingroup$
    @RobertIsrael for any arbitary numbers for $a_1, a_2,a_3,...a_k$, There's an always closed form for $a_n$. Do I understand correct?
    $endgroup$
    – Beginner
    Jan 8 at 12:46












  • $begingroup$
    @Beginner, it depends on what you consider a "closed form". That term is not really well-defined. In a sense you have given a "closed form" in your definition of the sequence.
    $endgroup$
    – Mees de Vries
    Jan 8 at 12:53










  • $begingroup$
    @MeesdeVries I tried to give 2 example for a good understanding of the question..
    $endgroup$
    – Beginner
    Jan 8 at 12:56










  • $begingroup$
    This might be a closed form for your first sequence: $$ a_k = begin{cases} 1&text{if $k = 1 mod4$,}\ 3&text{if $k = 2 mod4$,}\ 5&text{if $k = 3 mod4$,}\ 7&text{else.} end{cases} $$ Does that satisfy what you're looking for in a "closed form"?
    $endgroup$
    – Mees de Vries
    Jan 8 at 13:08
















2












$begingroup$


This question is a generalized form of the problem I asked before:



Algebraic Closed Form for $sum_{n=1}^{k}left( n- 3 lfloor frac{n-1}{3} rfloorright)$




Let, look at this periodic sequence:



$$a_n=left{a_1,a_2,a_3,a_4,a_5,cdots a_k ; a_1,a_2,a_3,a_4,a_5, cdots a_k; a_1,a_2,a_3,a_4,a_5,cdots a_k;cdots right}$$, where $left{a_1, a_2, a_3, cdots a_k right} in mathbb{Z^{+}}$ and $a_{k+1}=a_1, a_{k+2}=a_2, a_{k+3}=a_3, cdots a_{2k}=a_k, cdots$




For the sequence $a_k=left{ a_1,a_2,a_3,...,a_k right}$ , $k$ is a finite number. $a_1,a_2,...a_k$ are arbitary numbers.



Verbally, the series $a_n$ consists of an infinite number of periodic repetitions of the finite series $a_k.$




Finally my question is:



a) If there is an exist a algebraic closed form, for finite series $a_k$, in this case, does the $ a_n $ series always have a algebraic closed form?



b) If there is not an exist a algebraic closed form, for finite series $a_k$, in this case, does the $ a_n $ series always have a algebraic closed form?




I mean ,for example:




a)



$a_n=left{ 1,3,5,7,1,3,5,7,1,3,5,7,1,3,5,7,1,3,5,7cdotsright}$



b)



$a_n=left{1,8,2,6,5,9,1,8,2,6,5,9,1,8,2,6,5,9,1,8,2,6,5,9cdotsright}$




Thank you very much.










share|cite|improve this question











$endgroup$












  • $begingroup$
    A finite sequence always does have a closed form, and so does a periodic sequence.
    $endgroup$
    – Robert Israel
    Jan 8 at 12:39










  • $begingroup$
    @RobertIsrael for any arbitary numbers for $a_1, a_2,a_3,...a_k$, There's an always closed form for $a_n$. Do I understand correct?
    $endgroup$
    – Beginner
    Jan 8 at 12:46












  • $begingroup$
    @Beginner, it depends on what you consider a "closed form". That term is not really well-defined. In a sense you have given a "closed form" in your definition of the sequence.
    $endgroup$
    – Mees de Vries
    Jan 8 at 12:53










  • $begingroup$
    @MeesdeVries I tried to give 2 example for a good understanding of the question..
    $endgroup$
    – Beginner
    Jan 8 at 12:56










  • $begingroup$
    This might be a closed form for your first sequence: $$ a_k = begin{cases} 1&text{if $k = 1 mod4$,}\ 3&text{if $k = 2 mod4$,}\ 5&text{if $k = 3 mod4$,}\ 7&text{else.} end{cases} $$ Does that satisfy what you're looking for in a "closed form"?
    $endgroup$
    – Mees de Vries
    Jan 8 at 13:08














2












2








2





$begingroup$


This question is a generalized form of the problem I asked before:



Algebraic Closed Form for $sum_{n=1}^{k}left( n- 3 lfloor frac{n-1}{3} rfloorright)$




Let, look at this periodic sequence:



$$a_n=left{a_1,a_2,a_3,a_4,a_5,cdots a_k ; a_1,a_2,a_3,a_4,a_5, cdots a_k; a_1,a_2,a_3,a_4,a_5,cdots a_k;cdots right}$$, where $left{a_1, a_2, a_3, cdots a_k right} in mathbb{Z^{+}}$ and $a_{k+1}=a_1, a_{k+2}=a_2, a_{k+3}=a_3, cdots a_{2k}=a_k, cdots$




For the sequence $a_k=left{ a_1,a_2,a_3,...,a_k right}$ , $k$ is a finite number. $a_1,a_2,...a_k$ are arbitary numbers.



Verbally, the series $a_n$ consists of an infinite number of periodic repetitions of the finite series $a_k.$




Finally my question is:



a) If there is an exist a algebraic closed form, for finite series $a_k$, in this case, does the $ a_n $ series always have a algebraic closed form?



b) If there is not an exist a algebraic closed form, for finite series $a_k$, in this case, does the $ a_n $ series always have a algebraic closed form?




I mean ,for example:




a)



$a_n=left{ 1,3,5,7,1,3,5,7,1,3,5,7,1,3,5,7,1,3,5,7cdotsright}$



b)



$a_n=left{1,8,2,6,5,9,1,8,2,6,5,9,1,8,2,6,5,9,1,8,2,6,5,9cdotsright}$




Thank you very much.










share|cite|improve this question











$endgroup$




This question is a generalized form of the problem I asked before:



Algebraic Closed Form for $sum_{n=1}^{k}left( n- 3 lfloor frac{n-1}{3} rfloorright)$




Let, look at this periodic sequence:



$$a_n=left{a_1,a_2,a_3,a_4,a_5,cdots a_k ; a_1,a_2,a_3,a_4,a_5, cdots a_k; a_1,a_2,a_3,a_4,a_5,cdots a_k;cdots right}$$, where $left{a_1, a_2, a_3, cdots a_k right} in mathbb{Z^{+}}$ and $a_{k+1}=a_1, a_{k+2}=a_2, a_{k+3}=a_3, cdots a_{2k}=a_k, cdots$




For the sequence $a_k=left{ a_1,a_2,a_3,...,a_k right}$ , $k$ is a finite number. $a_1,a_2,...a_k$ are arbitary numbers.



Verbally, the series $a_n$ consists of an infinite number of periodic repetitions of the finite series $a_k.$




Finally my question is:



a) If there is an exist a algebraic closed form, for finite series $a_k$, in this case, does the $ a_n $ series always have a algebraic closed form?



b) If there is not an exist a algebraic closed form, for finite series $a_k$, in this case, does the $ a_n $ series always have a algebraic closed form?




I mean ,for example:




a)



$a_n=left{ 1,3,5,7,1,3,5,7,1,3,5,7,1,3,5,7,1,3,5,7cdotsright}$



b)



$a_n=left{1,8,2,6,5,9,1,8,2,6,5,9,1,8,2,6,5,9,1,8,2,6,5,9cdotsright}$




Thank you very much.







sequences-and-series soft-question closed-form






share|cite|improve this question















share|cite|improve this question













share|cite|improve this question




share|cite|improve this question








edited Jan 8 at 16:50







Beginner

















asked Jan 8 at 12:36









BeginnerBeginner

341110




341110












  • $begingroup$
    A finite sequence always does have a closed form, and so does a periodic sequence.
    $endgroup$
    – Robert Israel
    Jan 8 at 12:39










  • $begingroup$
    @RobertIsrael for any arbitary numbers for $a_1, a_2,a_3,...a_k$, There's an always closed form for $a_n$. Do I understand correct?
    $endgroup$
    – Beginner
    Jan 8 at 12:46












  • $begingroup$
    @Beginner, it depends on what you consider a "closed form". That term is not really well-defined. In a sense you have given a "closed form" in your definition of the sequence.
    $endgroup$
    – Mees de Vries
    Jan 8 at 12:53










  • $begingroup$
    @MeesdeVries I tried to give 2 example for a good understanding of the question..
    $endgroup$
    – Beginner
    Jan 8 at 12:56










  • $begingroup$
    This might be a closed form for your first sequence: $$ a_k = begin{cases} 1&text{if $k = 1 mod4$,}\ 3&text{if $k = 2 mod4$,}\ 5&text{if $k = 3 mod4$,}\ 7&text{else.} end{cases} $$ Does that satisfy what you're looking for in a "closed form"?
    $endgroup$
    – Mees de Vries
    Jan 8 at 13:08


















  • $begingroup$
    A finite sequence always does have a closed form, and so does a periodic sequence.
    $endgroup$
    – Robert Israel
    Jan 8 at 12:39










  • $begingroup$
    @RobertIsrael for any arbitary numbers for $a_1, a_2,a_3,...a_k$, There's an always closed form for $a_n$. Do I understand correct?
    $endgroup$
    – Beginner
    Jan 8 at 12:46












  • $begingroup$
    @Beginner, it depends on what you consider a "closed form". That term is not really well-defined. In a sense you have given a "closed form" in your definition of the sequence.
    $endgroup$
    – Mees de Vries
    Jan 8 at 12:53










  • $begingroup$
    @MeesdeVries I tried to give 2 example for a good understanding of the question..
    $endgroup$
    – Beginner
    Jan 8 at 12:56










  • $begingroup$
    This might be a closed form for your first sequence: $$ a_k = begin{cases} 1&text{if $k = 1 mod4$,}\ 3&text{if $k = 2 mod4$,}\ 5&text{if $k = 3 mod4$,}\ 7&text{else.} end{cases} $$ Does that satisfy what you're looking for in a "closed form"?
    $endgroup$
    – Mees de Vries
    Jan 8 at 13:08
















$begingroup$
A finite sequence always does have a closed form, and so does a periodic sequence.
$endgroup$
– Robert Israel
Jan 8 at 12:39




$begingroup$
A finite sequence always does have a closed form, and so does a periodic sequence.
$endgroup$
– Robert Israel
Jan 8 at 12:39












$begingroup$
@RobertIsrael for any arbitary numbers for $a_1, a_2,a_3,...a_k$, There's an always closed form for $a_n$. Do I understand correct?
$endgroup$
– Beginner
Jan 8 at 12:46






$begingroup$
@RobertIsrael for any arbitary numbers for $a_1, a_2,a_3,...a_k$, There's an always closed form for $a_n$. Do I understand correct?
$endgroup$
– Beginner
Jan 8 at 12:46














$begingroup$
@Beginner, it depends on what you consider a "closed form". That term is not really well-defined. In a sense you have given a "closed form" in your definition of the sequence.
$endgroup$
– Mees de Vries
Jan 8 at 12:53




$begingroup$
@Beginner, it depends on what you consider a "closed form". That term is not really well-defined. In a sense you have given a "closed form" in your definition of the sequence.
$endgroup$
– Mees de Vries
Jan 8 at 12:53












$begingroup$
@MeesdeVries I tried to give 2 example for a good understanding of the question..
$endgroup$
– Beginner
Jan 8 at 12:56




$begingroup$
@MeesdeVries I tried to give 2 example for a good understanding of the question..
$endgroup$
– Beginner
Jan 8 at 12:56












$begingroup$
This might be a closed form for your first sequence: $$ a_k = begin{cases} 1&text{if $k = 1 mod4$,}\ 3&text{if $k = 2 mod4$,}\ 5&text{if $k = 3 mod4$,}\ 7&text{else.} end{cases} $$ Does that satisfy what you're looking for in a "closed form"?
$endgroup$
– Mees de Vries
Jan 8 at 13:08




$begingroup$
This might be a closed form for your first sequence: $$ a_k = begin{cases} 1&text{if $k = 1 mod4$,}\ 3&text{if $k = 2 mod4$,}\ 5&text{if $k = 3 mod4$,}\ 7&text{else.} end{cases} $$ Does that satisfy what you're looking for in a "closed form"?
$endgroup$
– Mees de Vries
Jan 8 at 13:08










2 Answers
2






active

oldest

votes


















2












$begingroup$

A periodic sequence with period $P$ can always be written as a trigonometric polynomial $$a_n = sum_{j=0}^{P-1} b_j cos(2 pi j n/P) + sum_{j=1}^{P-1} c_j sin(2 pi j n/P)$$
for some coefficients $b_j$ and $c_j$ (look up Finite Fourier Transform).



Thus your first example can be written as
$$a_n = 4-cos left( pi,j/2 right) -cos left( pi,j right) -cos
left( 3,pi,j/2 right) -sin left( pi,j/2 right) +sin
left( 3,pi,j/2 right)
$$






share|cite|improve this answer











$endgroup$





















    2












    $begingroup$

    Let $f(x)$ be a polynomial interpolating $a_1,a_2,a_3,dots,a_k$.



    Then the sequence is given by $a_n = f(1+((n-1) bmod k))$.



    If you can't use mod directly, but can use floor, then note that
    $$
    a bmod b = a - b leftlfloor dfrac{a}{b} rightrfloor
    $$

    for $a,b in mathbb N$.






    share|cite|improve this answer











    $endgroup$













      Your Answer





      StackExchange.ifUsing("editor", function () {
      return StackExchange.using("mathjaxEditing", function () {
      StackExchange.MarkdownEditor.creationCallbacks.add(function (editor, postfix) {
      StackExchange.mathjaxEditing.prepareWmdForMathJax(editor, postfix, [["$", "$"], ["\\(","\\)"]]);
      });
      });
      }, "mathjax-editing");

      StackExchange.ready(function() {
      var channelOptions = {
      tags: "".split(" "),
      id: "69"
      };
      initTagRenderer("".split(" "), "".split(" "), channelOptions);

      StackExchange.using("externalEditor", function() {
      // Have to fire editor after snippets, if snippets enabled
      if (StackExchange.settings.snippets.snippetsEnabled) {
      StackExchange.using("snippets", function() {
      createEditor();
      });
      }
      else {
      createEditor();
      }
      });

      function createEditor() {
      StackExchange.prepareEditor({
      heartbeatType: 'answer',
      autoActivateHeartbeat: false,
      convertImagesToLinks: true,
      noModals: true,
      showLowRepImageUploadWarning: true,
      reputationToPostImages: 10,
      bindNavPrevention: true,
      postfix: "",
      imageUploader: {
      brandingHtml: "Powered by u003ca class="icon-imgur-white" href="https://imgur.com/"u003eu003c/au003e",
      contentPolicyHtml: "User contributions licensed under u003ca href="https://creativecommons.org/licenses/by-sa/3.0/"u003ecc by-sa 3.0 with attribution requiredu003c/au003e u003ca href="https://stackoverflow.com/legal/content-policy"u003e(content policy)u003c/au003e",
      allowUrls: true
      },
      noCode: true, onDemand: true,
      discardSelector: ".discard-answer"
      ,immediatelyShowMarkdownHelp:true
      });


      }
      });














      draft saved

      draft discarded


















      StackExchange.ready(
      function () {
      StackExchange.openid.initPostLogin('.new-post-login', 'https%3a%2f%2fmath.stackexchange.com%2fquestions%2f3066118%2ffor-the-periodic-sequence-is-there-always-an-algebraic-closed-form%23new-answer', 'question_page');
      }
      );

      Post as a guest















      Required, but never shown

























      2 Answers
      2






      active

      oldest

      votes








      2 Answers
      2






      active

      oldest

      votes









      active

      oldest

      votes






      active

      oldest

      votes









      2












      $begingroup$

      A periodic sequence with period $P$ can always be written as a trigonometric polynomial $$a_n = sum_{j=0}^{P-1} b_j cos(2 pi j n/P) + sum_{j=1}^{P-1} c_j sin(2 pi j n/P)$$
      for some coefficients $b_j$ and $c_j$ (look up Finite Fourier Transform).



      Thus your first example can be written as
      $$a_n = 4-cos left( pi,j/2 right) -cos left( pi,j right) -cos
      left( 3,pi,j/2 right) -sin left( pi,j/2 right) +sin
      left( 3,pi,j/2 right)
      $$






      share|cite|improve this answer











      $endgroup$


















        2












        $begingroup$

        A periodic sequence with period $P$ can always be written as a trigonometric polynomial $$a_n = sum_{j=0}^{P-1} b_j cos(2 pi j n/P) + sum_{j=1}^{P-1} c_j sin(2 pi j n/P)$$
        for some coefficients $b_j$ and $c_j$ (look up Finite Fourier Transform).



        Thus your first example can be written as
        $$a_n = 4-cos left( pi,j/2 right) -cos left( pi,j right) -cos
        left( 3,pi,j/2 right) -sin left( pi,j/2 right) +sin
        left( 3,pi,j/2 right)
        $$






        share|cite|improve this answer











        $endgroup$
















          2












          2








          2





          $begingroup$

          A periodic sequence with period $P$ can always be written as a trigonometric polynomial $$a_n = sum_{j=0}^{P-1} b_j cos(2 pi j n/P) + sum_{j=1}^{P-1} c_j sin(2 pi j n/P)$$
          for some coefficients $b_j$ and $c_j$ (look up Finite Fourier Transform).



          Thus your first example can be written as
          $$a_n = 4-cos left( pi,j/2 right) -cos left( pi,j right) -cos
          left( 3,pi,j/2 right) -sin left( pi,j/2 right) +sin
          left( 3,pi,j/2 right)
          $$






          share|cite|improve this answer











          $endgroup$



          A periodic sequence with period $P$ can always be written as a trigonometric polynomial $$a_n = sum_{j=0}^{P-1} b_j cos(2 pi j n/P) + sum_{j=1}^{P-1} c_j sin(2 pi j n/P)$$
          for some coefficients $b_j$ and $c_j$ (look up Finite Fourier Transform).



          Thus your first example can be written as
          $$a_n = 4-cos left( pi,j/2 right) -cos left( pi,j right) -cos
          left( 3,pi,j/2 right) -sin left( pi,j/2 right) +sin
          left( 3,pi,j/2 right)
          $$







          share|cite|improve this answer














          share|cite|improve this answer



          share|cite|improve this answer








          edited Jan 8 at 16:59

























          answered Jan 8 at 16:39









          Robert IsraelRobert Israel

          321k23210462




          321k23210462























              2












              $begingroup$

              Let $f(x)$ be a polynomial interpolating $a_1,a_2,a_3,dots,a_k$.



              Then the sequence is given by $a_n = f(1+((n-1) bmod k))$.



              If you can't use mod directly, but can use floor, then note that
              $$
              a bmod b = a - b leftlfloor dfrac{a}{b} rightrfloor
              $$

              for $a,b in mathbb N$.






              share|cite|improve this answer











              $endgroup$


















                2












                $begingroup$

                Let $f(x)$ be a polynomial interpolating $a_1,a_2,a_3,dots,a_k$.



                Then the sequence is given by $a_n = f(1+((n-1) bmod k))$.



                If you can't use mod directly, but can use floor, then note that
                $$
                a bmod b = a - b leftlfloor dfrac{a}{b} rightrfloor
                $$

                for $a,b in mathbb N$.






                share|cite|improve this answer











                $endgroup$
















                  2












                  2








                  2





                  $begingroup$

                  Let $f(x)$ be a polynomial interpolating $a_1,a_2,a_3,dots,a_k$.



                  Then the sequence is given by $a_n = f(1+((n-1) bmod k))$.



                  If you can't use mod directly, but can use floor, then note that
                  $$
                  a bmod b = a - b leftlfloor dfrac{a}{b} rightrfloor
                  $$

                  for $a,b in mathbb N$.






                  share|cite|improve this answer











                  $endgroup$



                  Let $f(x)$ be a polynomial interpolating $a_1,a_2,a_3,dots,a_k$.



                  Then the sequence is given by $a_n = f(1+((n-1) bmod k))$.



                  If you can't use mod directly, but can use floor, then note that
                  $$
                  a bmod b = a - b leftlfloor dfrac{a}{b} rightrfloor
                  $$

                  for $a,b in mathbb N$.







                  share|cite|improve this answer














                  share|cite|improve this answer



                  share|cite|improve this answer








                  edited Jan 8 at 15:13

























                  answered Jan 8 at 13:08









                  lhflhf

                  164k10170395




                  164k10170395






























                      draft saved

                      draft discarded




















































                      Thanks for contributing an answer to Mathematics Stack Exchange!


                      • Please be sure to answer the question. Provide details and share your research!

                      But avoid



                      • Asking for help, clarification, or responding to other answers.

                      • Making statements based on opinion; back them up with references or personal experience.


                      Use MathJax to format equations. MathJax reference.


                      To learn more, see our tips on writing great answers.




                      draft saved


                      draft discarded














                      StackExchange.ready(
                      function () {
                      StackExchange.openid.initPostLogin('.new-post-login', 'https%3a%2f%2fmath.stackexchange.com%2fquestions%2f3066118%2ffor-the-periodic-sequence-is-there-always-an-algebraic-closed-form%23new-answer', 'question_page');
                      }
                      );

                      Post as a guest















                      Required, but never shown





















































                      Required, but never shown














                      Required, but never shown












                      Required, but never shown







                      Required, but never shown

































                      Required, but never shown














                      Required, but never shown












                      Required, but never shown







                      Required, but never shown







                      Popular posts from this blog

                      An IMO inspired problem

                      Management

                      Has there ever been an instance of an active nuclear power plant within or near a war zone?